- Mon Sep 04, 2017 8:15 pm
#39212
Complete Question Explanation
(The complete setup for this game can be found here: lsat/viewtopic.php?t=12911)
The correct answer choice is (A)
If one of the rugs is solid Y, we can safely rule out the 2-2-1 distribution, which requires Y to be used in a multicolored rug together with F or T. The 3-1-1 distribution therefore applies:
It is difficult to make any additional deductions here, nor do we have to. Indeed, before you do any more work, take a look at the five answer choices. Identifying the proper distribution immediately proves answer choice (A) to be false, because there must be exactly two solid color rugs, not one. Therefore, answer choice (A) is correct.
If you are running short on time, you can safely decide to skip verifying the remaining four answer choices. Since the incorrect answer choices are all “could be true” statements, eliminating them would be a laborious task, potentially requiring the use of local diagrams to test each possibility. Prudence is certainly a virtue; lack of confidence is not.
Answer choice (A) is the correct answer choice.
Answer choice (B) is incorrect, because one of the rugs could be solid F:
Answer choice (C) is incorrect, because T could be used in one of the rugs. For instance:
Answer choice (D) is incorrect, because F and O could be used together in the multicolored rug, as shown in the setup for answer choice (C).
Answer choice (E) is incorrect, because P and W could be used together in the multicolored rug, as shown in the setup for answer choice (B).
(The complete setup for this game can be found here: lsat/viewtopic.php?t=12911)
The correct answer choice is (A)
If one of the rugs is solid Y, we can safely rule out the 2-2-1 distribution, which requires Y to be used in a multicolored rug together with F or T. The 3-1-1 distribution therefore applies:
It is difficult to make any additional deductions here, nor do we have to. Indeed, before you do any more work, take a look at the five answer choices. Identifying the proper distribution immediately proves answer choice (A) to be false, because there must be exactly two solid color rugs, not one. Therefore, answer choice (A) is correct.
If you are running short on time, you can safely decide to skip verifying the remaining four answer choices. Since the incorrect answer choices are all “could be true” statements, eliminating them would be a laborious task, potentially requiring the use of local diagrams to test each possibility. Prudence is certainly a virtue; lack of confidence is not.
Answer choice (A) is the correct answer choice.
Answer choice (B) is incorrect, because one of the rugs could be solid F:
Answer choice (C) is incorrect, because T could be used in one of the rugs. For instance:
Answer choice (D) is incorrect, because F and O could be used together in the multicolored rug, as shown in the setup for answer choice (C).
Answer choice (E) is incorrect, because P and W could be used together in the multicolored rug, as shown in the setup for answer choice (B).
You do not have the required permissions to view the files attached to this post.